Let $B(t)$ be a Brownian Motion. Show that the following processes are Brownian Motions on $[0,T].$












0












$begingroup$


1) $X(t)=-B(t).$



Since $-X(t)$ is a Gaussian Process with zero mean, $X(t)$ is also Gaussian Process with zero mean. We simply want to show that its covariance function is $min(t,s).$ This is trivial since $$gamma(s,t) = text{cov}(-B(t)times -B(s)) = text{cov}(B(t)B(s))=min(t,s).$$ Thus $X(t) = -B(t)$ is also a Brownian motion.



2) $X(t) = B(T-t)-B(T).$



I am not sure how to argue why $X(t)$ has zero mean. However for the covariance function we have:
$$gamma(s,t) = text{cov}((B(T-s)-B(s))times (B(T-t)-B(t)))$$
$$= E[(B(T-s)-B(s))times (B(T-t)-B(t))] $$
$$= max(t,s)-min(T-s,t)-min(T-t,s)+min(s,t)$$
$$= t + s - 2min(T,s+t).$$
I don't know how to show that this is the same as $min(s,t).$



3) $X(t) = cB(t/c^2)$ where $Tleq infty.$



For this I can show that $gamma(s,t) = text{cov}(X(s),X(t))$
$$ = c^2E(B(s/c^2)B(t/c^2)) = c^2 min(s/c^2,t/c^2)=min(s,t).$$
However, I am not sure how to show that $X(t)$ is a Gaussian process with mean $0.$



4) $X(t) = tB(1/t)$ and $X(0)=0.$



For this I can show that $gamma(s,t) = text{cov}(X(s),X(t))$
$$ = tsE(B(1/s)B(1/t)) = ts min(1/t,1/s)=min(s,t).$$
However, for this part too I am not sure how to show that $X(t)$ is a Gaussian process with mean $0.$










share|cite|improve this question









$endgroup$

















    0












    $begingroup$


    1) $X(t)=-B(t).$



    Since $-X(t)$ is a Gaussian Process with zero mean, $X(t)$ is also Gaussian Process with zero mean. We simply want to show that its covariance function is $min(t,s).$ This is trivial since $$gamma(s,t) = text{cov}(-B(t)times -B(s)) = text{cov}(B(t)B(s))=min(t,s).$$ Thus $X(t) = -B(t)$ is also a Brownian motion.



    2) $X(t) = B(T-t)-B(T).$



    I am not sure how to argue why $X(t)$ has zero mean. However for the covariance function we have:
    $$gamma(s,t) = text{cov}((B(T-s)-B(s))times (B(T-t)-B(t)))$$
    $$= E[(B(T-s)-B(s))times (B(T-t)-B(t))] $$
    $$= max(t,s)-min(T-s,t)-min(T-t,s)+min(s,t)$$
    $$= t + s - 2min(T,s+t).$$
    I don't know how to show that this is the same as $min(s,t).$



    3) $X(t) = cB(t/c^2)$ where $Tleq infty.$



    For this I can show that $gamma(s,t) = text{cov}(X(s),X(t))$
    $$ = c^2E(B(s/c^2)B(t/c^2)) = c^2 min(s/c^2,t/c^2)=min(s,t).$$
    However, I am not sure how to show that $X(t)$ is a Gaussian process with mean $0.$



    4) $X(t) = tB(1/t)$ and $X(0)=0.$



    For this I can show that $gamma(s,t) = text{cov}(X(s),X(t))$
    $$ = tsE(B(1/s)B(1/t)) = ts min(1/t,1/s)=min(s,t).$$
    However, for this part too I am not sure how to show that $X(t)$ is a Gaussian process with mean $0.$










    share|cite|improve this question









    $endgroup$















      0












      0








      0





      $begingroup$


      1) $X(t)=-B(t).$



      Since $-X(t)$ is a Gaussian Process with zero mean, $X(t)$ is also Gaussian Process with zero mean. We simply want to show that its covariance function is $min(t,s).$ This is trivial since $$gamma(s,t) = text{cov}(-B(t)times -B(s)) = text{cov}(B(t)B(s))=min(t,s).$$ Thus $X(t) = -B(t)$ is also a Brownian motion.



      2) $X(t) = B(T-t)-B(T).$



      I am not sure how to argue why $X(t)$ has zero mean. However for the covariance function we have:
      $$gamma(s,t) = text{cov}((B(T-s)-B(s))times (B(T-t)-B(t)))$$
      $$= E[(B(T-s)-B(s))times (B(T-t)-B(t))] $$
      $$= max(t,s)-min(T-s,t)-min(T-t,s)+min(s,t)$$
      $$= t + s - 2min(T,s+t).$$
      I don't know how to show that this is the same as $min(s,t).$



      3) $X(t) = cB(t/c^2)$ where $Tleq infty.$



      For this I can show that $gamma(s,t) = text{cov}(X(s),X(t))$
      $$ = c^2E(B(s/c^2)B(t/c^2)) = c^2 min(s/c^2,t/c^2)=min(s,t).$$
      However, I am not sure how to show that $X(t)$ is a Gaussian process with mean $0.$



      4) $X(t) = tB(1/t)$ and $X(0)=0.$



      For this I can show that $gamma(s,t) = text{cov}(X(s),X(t))$
      $$ = tsE(B(1/s)B(1/t)) = ts min(1/t,1/s)=min(s,t).$$
      However, for this part too I am not sure how to show that $X(t)$ is a Gaussian process with mean $0.$










      share|cite|improve this question









      $endgroup$




      1) $X(t)=-B(t).$



      Since $-X(t)$ is a Gaussian Process with zero mean, $X(t)$ is also Gaussian Process with zero mean. We simply want to show that its covariance function is $min(t,s).$ This is trivial since $$gamma(s,t) = text{cov}(-B(t)times -B(s)) = text{cov}(B(t)B(s))=min(t,s).$$ Thus $X(t) = -B(t)$ is also a Brownian motion.



      2) $X(t) = B(T-t)-B(T).$



      I am not sure how to argue why $X(t)$ has zero mean. However for the covariance function we have:
      $$gamma(s,t) = text{cov}((B(T-s)-B(s))times (B(T-t)-B(t)))$$
      $$= E[(B(T-s)-B(s))times (B(T-t)-B(t))] $$
      $$= max(t,s)-min(T-s,t)-min(T-t,s)+min(s,t)$$
      $$= t + s - 2min(T,s+t).$$
      I don't know how to show that this is the same as $min(s,t).$



      3) $X(t) = cB(t/c^2)$ where $Tleq infty.$



      For this I can show that $gamma(s,t) = text{cov}(X(s),X(t))$
      $$ = c^2E(B(s/c^2)B(t/c^2)) = c^2 min(s/c^2,t/c^2)=min(s,t).$$
      However, I am not sure how to show that $X(t)$ is a Gaussian process with mean $0.$



      4) $X(t) = tB(1/t)$ and $X(0)=0.$



      For this I can show that $gamma(s,t) = text{cov}(X(s),X(t))$
      $$ = tsE(B(1/s)B(1/t)) = ts min(1/t,1/s)=min(s,t).$$
      However, for this part too I am not sure how to show that $X(t)$ is a Gaussian process with mean $0.$







      probability






      share|cite|improve this question













      share|cite|improve this question











      share|cite|improve this question




      share|cite|improve this question










      asked Dec 18 '18 at 21:12









      Hello_WorldHello_World

      4,13121831




      4,13121831






















          1 Answer
          1






          active

          oldest

          votes


















          0












          $begingroup$

          In (2), you get directly $E[X(t)] = E[B(T-t)] - E[B(T)] = 0$ because $B$ is a centered Gaussian process. For the covariance, you made a small error. You want to calculate
          $$
          Cov(X(t),X(s)) = Cov(B(T-t)-B(T), B(T-s)-B(T)).
          $$

          You should also mention that $X(0) = 0$, which is trivial of course.



          In (3), you get similarly that $X(t)$ has mean $0$.Let's show that $X$ is a Gaussian process. To this end, let $0 leq t_1 < dots t_n leq T$. We have
          $$
          (X(t_1),dots,X(t_n)) = c(B(t_1/c^2),dots,B(t_n/c^2)).
          $$

          The right hand side has a multivariate normal distribution because $B$ is a Gaussian process.



          For (4), you need that $B$ is a Brownian Motion on $[0,infty)$ (not on a finite interval $[0,T]$). Using the same agumentation as above, you get that $X$ is Gaussion with mean $0$.



          In all cases, you also need to show that $X$ is continuous. (This is part of the definition of a Brownian Motion.) This is trivial for (1)-(3). In (4), however, the continuity in $0$ is not trivial.






          share|cite|improve this answer











          $endgroup$













            Your Answer





            StackExchange.ifUsing("editor", function () {
            return StackExchange.using("mathjaxEditing", function () {
            StackExchange.MarkdownEditor.creationCallbacks.add(function (editor, postfix) {
            StackExchange.mathjaxEditing.prepareWmdForMathJax(editor, postfix, [["$", "$"], ["\\(","\\)"]]);
            });
            });
            }, "mathjax-editing");

            StackExchange.ready(function() {
            var channelOptions = {
            tags: "".split(" "),
            id: "69"
            };
            initTagRenderer("".split(" "), "".split(" "), channelOptions);

            StackExchange.using("externalEditor", function() {
            // Have to fire editor after snippets, if snippets enabled
            if (StackExchange.settings.snippets.snippetsEnabled) {
            StackExchange.using("snippets", function() {
            createEditor();
            });
            }
            else {
            createEditor();
            }
            });

            function createEditor() {
            StackExchange.prepareEditor({
            heartbeatType: 'answer',
            autoActivateHeartbeat: false,
            convertImagesToLinks: true,
            noModals: true,
            showLowRepImageUploadWarning: true,
            reputationToPostImages: 10,
            bindNavPrevention: true,
            postfix: "",
            imageUploader: {
            brandingHtml: "Powered by u003ca class="icon-imgur-white" href="https://imgur.com/"u003eu003c/au003e",
            contentPolicyHtml: "User contributions licensed under u003ca href="https://creativecommons.org/licenses/by-sa/3.0/"u003ecc by-sa 3.0 with attribution requiredu003c/au003e u003ca href="https://stackoverflow.com/legal/content-policy"u003e(content policy)u003c/au003e",
            allowUrls: true
            },
            noCode: true, onDemand: true,
            discardSelector: ".discard-answer"
            ,immediatelyShowMarkdownHelp:true
            });


            }
            });














            draft saved

            draft discarded


















            StackExchange.ready(
            function () {
            StackExchange.openid.initPostLogin('.new-post-login', 'https%3a%2f%2fmath.stackexchange.com%2fquestions%2f3045702%2flet-bt-be-a-brownian-motion-show-that-the-following-processes-are-brownian%23new-answer', 'question_page');
            }
            );

            Post as a guest















            Required, but never shown

























            1 Answer
            1






            active

            oldest

            votes








            1 Answer
            1






            active

            oldest

            votes









            active

            oldest

            votes






            active

            oldest

            votes









            0












            $begingroup$

            In (2), you get directly $E[X(t)] = E[B(T-t)] - E[B(T)] = 0$ because $B$ is a centered Gaussian process. For the covariance, you made a small error. You want to calculate
            $$
            Cov(X(t),X(s)) = Cov(B(T-t)-B(T), B(T-s)-B(T)).
            $$

            You should also mention that $X(0) = 0$, which is trivial of course.



            In (3), you get similarly that $X(t)$ has mean $0$.Let's show that $X$ is a Gaussian process. To this end, let $0 leq t_1 < dots t_n leq T$. We have
            $$
            (X(t_1),dots,X(t_n)) = c(B(t_1/c^2),dots,B(t_n/c^2)).
            $$

            The right hand side has a multivariate normal distribution because $B$ is a Gaussian process.



            For (4), you need that $B$ is a Brownian Motion on $[0,infty)$ (not on a finite interval $[0,T]$). Using the same agumentation as above, you get that $X$ is Gaussion with mean $0$.



            In all cases, you also need to show that $X$ is continuous. (This is part of the definition of a Brownian Motion.) This is trivial for (1)-(3). In (4), however, the continuity in $0$ is not trivial.






            share|cite|improve this answer











            $endgroup$


















              0












              $begingroup$

              In (2), you get directly $E[X(t)] = E[B(T-t)] - E[B(T)] = 0$ because $B$ is a centered Gaussian process. For the covariance, you made a small error. You want to calculate
              $$
              Cov(X(t),X(s)) = Cov(B(T-t)-B(T), B(T-s)-B(T)).
              $$

              You should also mention that $X(0) = 0$, which is trivial of course.



              In (3), you get similarly that $X(t)$ has mean $0$.Let's show that $X$ is a Gaussian process. To this end, let $0 leq t_1 < dots t_n leq T$. We have
              $$
              (X(t_1),dots,X(t_n)) = c(B(t_1/c^2),dots,B(t_n/c^2)).
              $$

              The right hand side has a multivariate normal distribution because $B$ is a Gaussian process.



              For (4), you need that $B$ is a Brownian Motion on $[0,infty)$ (not on a finite interval $[0,T]$). Using the same agumentation as above, you get that $X$ is Gaussion with mean $0$.



              In all cases, you also need to show that $X$ is continuous. (This is part of the definition of a Brownian Motion.) This is trivial for (1)-(3). In (4), however, the continuity in $0$ is not trivial.






              share|cite|improve this answer











              $endgroup$
















                0












                0








                0





                $begingroup$

                In (2), you get directly $E[X(t)] = E[B(T-t)] - E[B(T)] = 0$ because $B$ is a centered Gaussian process. For the covariance, you made a small error. You want to calculate
                $$
                Cov(X(t),X(s)) = Cov(B(T-t)-B(T), B(T-s)-B(T)).
                $$

                You should also mention that $X(0) = 0$, which is trivial of course.



                In (3), you get similarly that $X(t)$ has mean $0$.Let's show that $X$ is a Gaussian process. To this end, let $0 leq t_1 < dots t_n leq T$. We have
                $$
                (X(t_1),dots,X(t_n)) = c(B(t_1/c^2),dots,B(t_n/c^2)).
                $$

                The right hand side has a multivariate normal distribution because $B$ is a Gaussian process.



                For (4), you need that $B$ is a Brownian Motion on $[0,infty)$ (not on a finite interval $[0,T]$). Using the same agumentation as above, you get that $X$ is Gaussion with mean $0$.



                In all cases, you also need to show that $X$ is continuous. (This is part of the definition of a Brownian Motion.) This is trivial for (1)-(3). In (4), however, the continuity in $0$ is not trivial.






                share|cite|improve this answer











                $endgroup$



                In (2), you get directly $E[X(t)] = E[B(T-t)] - E[B(T)] = 0$ because $B$ is a centered Gaussian process. For the covariance, you made a small error. You want to calculate
                $$
                Cov(X(t),X(s)) = Cov(B(T-t)-B(T), B(T-s)-B(T)).
                $$

                You should also mention that $X(0) = 0$, which is trivial of course.



                In (3), you get similarly that $X(t)$ has mean $0$.Let's show that $X$ is a Gaussian process. To this end, let $0 leq t_1 < dots t_n leq T$. We have
                $$
                (X(t_1),dots,X(t_n)) = c(B(t_1/c^2),dots,B(t_n/c^2)).
                $$

                The right hand side has a multivariate normal distribution because $B$ is a Gaussian process.



                For (4), you need that $B$ is a Brownian Motion on $[0,infty)$ (not on a finite interval $[0,T]$). Using the same agumentation as above, you get that $X$ is Gaussion with mean $0$.



                In all cases, you also need to show that $X$ is continuous. (This is part of the definition of a Brownian Motion.) This is trivial for (1)-(3). In (4), however, the continuity in $0$ is not trivial.







                share|cite|improve this answer














                share|cite|improve this answer



                share|cite|improve this answer








                edited Dec 19 '18 at 14:26

























                answered Dec 19 '18 at 14:17









                Tki DenebTki Deneb

                32710




                32710






























                    draft saved

                    draft discarded




















































                    Thanks for contributing an answer to Mathematics Stack Exchange!


                    • Please be sure to answer the question. Provide details and share your research!

                    But avoid



                    • Asking for help, clarification, or responding to other answers.

                    • Making statements based on opinion; back them up with references or personal experience.


                    Use MathJax to format equations. MathJax reference.


                    To learn more, see our tips on writing great answers.




                    draft saved


                    draft discarded














                    StackExchange.ready(
                    function () {
                    StackExchange.openid.initPostLogin('.new-post-login', 'https%3a%2f%2fmath.stackexchange.com%2fquestions%2f3045702%2flet-bt-be-a-brownian-motion-show-that-the-following-processes-are-brownian%23new-answer', 'question_page');
                    }
                    );

                    Post as a guest















                    Required, but never shown





















































                    Required, but never shown














                    Required, but never shown












                    Required, but never shown







                    Required, but never shown

































                    Required, but never shown














                    Required, but never shown












                    Required, but never shown







                    Required, but never shown







                    Popular posts from this blog

                    To store a contact into the json file from server.js file using a class in NodeJS

                    Redirect URL with Chrome Remote Debugging Android Devices

                    Dieringhausen